Search found 9 matches


I would second that A weakens the conclusion the most. If there was another entrance to the cage that wasn't blocked until 1000 later, then it is impossible to properly identify the date of the paintings. Therefore, the Conclusion is weakened. The answer choice E states that identification was "...

by wiut

Wed Sep 18, 2013 8:38 am
Forum: Critical Reasoning
Topic: Dating!
Replies: 4
Views: 1672

But, x absolute could equal y absolute with any integer (one is obviously negative), say 3 and -3, 4 and -4. From (2) we can not infer the answer. (Look at your numbers! In each case they add up to zero.) Now X^Y=4 (I hope you understood that ^ means degree, sorry I don't know how to put symbols in...

by wiut

Mon Jan 10, 2011 9:54 am
Forum: Data Sufficiency
Topic: weird question from private tutor)
Replies: 13
Views: 8879

another weird

A number of eggs dyed various colors were hidden for an egg hunt. How many eggs in total were hidden?

1. The number of red eggs hidden was the square of an integer, while the total number of eggs hidden was 24 times that integer.
2. Exactly 143 of the eggs hidden were not red.

by wiut

Mon Jan 10, 2011 9:51 am
Forum: Data Sufficiency
Topic: another weird
Replies: 1
Views: 856

my answer is D (1) the only pair of x,y such that y>x and x^y=4, is x=-2, y=2, -2+2=0 suff (2)|x|=|y| if x>0 and y>0 then x=y but according to the problem y>x not possible if x<0 and y>0 then -x=y and we can find the sum x+(-x)=0 if x>0 and y<0 then x>y not possible if x<0 and y<0 then -x=-y x=y ag...

by wiut

Mon Jan 10, 2011 9:46 am
Forum: Data Sufficiency
Topic: weird question from private tutor)
Replies: 13
Views: 8879

weird question from private tutor)

if x and y are integers and x<y, what is the value of x+y?

1. x^y=4
2. x absolute = y absolute

it seems weird as obviously the only y that there is not Y which is less than X that satisfies either/both 1 and 2.....


what is your advise?
[/spoiler][/list]

by wiut

Mon Jan 10, 2011 9:23 am
Forum: Data Sufficiency
Topic: weird question from private tutor)
Replies: 13
Views: 8879

I saw this question yesterday on Grockit. I chose E, which turned to be the correct answer. My grounding for E was that it's the only choice that correctly (from my point of view) defines THE FIGHT and not the Speed Limit to be a problem....maybe I was just lucky, because after looking through the o...

by wiut

Mon Jan 10, 2011 6:04 am
Forum: Sentence Correction
Topic: Speed limit - long one!
Replies: 282
Views: 109896

First of all, thanks to the Experts (and non-experts) for such a fast reply! Its fantastic! I never actually thought that it could be that fast! :!: The explanation is perfect! Actually, I just made an error with (t-200,000) - I used (200,000*0,004x). Once again, many thanks and good working week fo...

by wiut

Mon Jan 10, 2011 4:27 am
Forum: Problem Solving
Topic: question from Grockit - can't get it
Replies: 4
Views: 1462

question from Grockit - can't get it

Ann and Bill are both salespeople at a car dealership. Ann earns $1250 each month plus 0.4 percent of her sales over $200,000 for that month. Bill earns 0.5 percent of his total sales each month. If in a given month Ann and Bill earned the same amount of money and had the same total sales, what was ...

by wiut

Mon Jan 10, 2011 3:57 am
Forum: Problem Solving
Topic: question from Grockit - can't get it
Replies: 4
Views: 1462

D looks like the right answer - it clearly and logically explains the Fight is regarded this and that. Unlike A, which basically has the same meaning, D is more straightforward.

Will the author provide us with the right answer anyway?

by wiut

Mon Jan 10, 2011 3:19 am
Forum: Sentence Correction
Topic: Speed limit - long one!
Replies: 282
Views: 109896